subject
Mathematics, 20.06.2021 03:10 Samuelmoreno4496

A customer who intends to purchase an appliance has three coupons, only one of which may be used:
Coupon 1: 15% off the listed price if the listed price is at least $100
Coupon 2: $30 off the listed price if the listed price is at least $200
Coupon 3: 20% off the amount by which the listed price exceeds $200
For which of the following listed prices will coupon I not offer a greater price
reduction than either coupon 2 or coupon 3?
A) $279.95 (B) $399.95 (C) $800.99
(C) $800.99 (D) $739.95 (E) $659.95

ansver
Answers: 2

Another question on Mathematics

question
Mathematics, 21.06.2019 15:30
David must install fencing around a lot that is shaped like a right angle.the side of lot that runs east-west is 200 ft long.the side of the lot runs north-south is 125 ft long.calculate how many feet of fencing he will need to surround the entire lot.
Answers: 2
question
Mathematics, 21.06.2019 18:00
Arecipe calls for 32 fluid ounces of heavy cream.how many 1 pint containers of heavy cream are needed to make the recipe?
Answers: 2
question
Mathematics, 21.06.2019 23:00
Which of the following graphs could represent a cubic function?
Answers: 1
question
Mathematics, 21.06.2019 23:00
The coordinate grid shows a pentagon. the pentagon is translated 3 units to the right and 5 units up to create a new pentagon. what is the rule for the location of the new pentagon?
Answers: 3
You know the right answer?
A customer who intends to purchase an appliance has three coupons, only one of which may be used:
Questions
question
Mathematics, 23.09.2019 14:50
question
Mathematics, 23.09.2019 14:50
Questions on the website: 13722367